[Toán 10]Bdt

Status
Không mở trả lời sau này.
I

ivory

[TEX]\fbox{\red Lasvia\: 2002[/TEX]

[TEX]\blue a,b,c,d [/TEX]là các số thực dương thoả mãn :

[TEX]\blue \sum_{a,b,c,d} \frac{1}{a^4+1}=1[/TEX].

Chứng minh:

[TEX]\fbox{ \blue abcd \ge 3[/TEX]
Lượng giác(hojoo lee)
[tex]a^2=tanx,b^2=tan^2y,c^2=tanz,d^2=tant[/TEX]
co [TEX]\cos^2x+cos^2y+cos^2z+cos^2t=1[/TEX]
*[TEX]sin^2x=cos^2y+cos^2z+cos^2t\ge 3\sqrt[3]{(cosycoszcost)^2}[/TEX]
*[TEX]sin^2y\ge 3\sqrt[3]{(cosxcoszcost)^2}[/TEX]
*[TEX]sin^2z\ge 3\sqrt[3]{(cosxcosycost)^2}[/TEX]
*[TEX]sin^2t\ge 3\sqrt[3]{(cosxcosycosz)^2}[/TEX]
 
Last edited by a moderator:
H

herrycuong_boy94

Tìm max
gif.latex
 
S

sieuthamtu_sieudaochit

Cho a,b,c la do dai ba canh tam giac. Chung minh:
[TEX]\frac{(b+c-a)^4}{a(a+b-c)}+\frac{(c+a-b)^4}{b(b+c-a)}+\frac{(a+b-c)^4}{c(c+a-b)}\geq ab+bc+ca[/TEX].
By Nguyen Van Huyen
Sử dụng bất đẳng thức Cauchy-Schwarz tz có
[TEX]\sum{\frac{(b+c-a)^4}{a(a+b-c)}}\ge\frac{(\sum{(b+c-a)}^2)^2}{a^2+b^2+c^2}[/TEX]
Dễ dàng chứng minh được rằng
[TEX]\sum{(b+c-a)}^2\ge a^2+b^2+c^2[/TEX]
Từ đó ta có [TEX]LHS \ge a^2+b^2+c^2\ge ab+bc+ca[/TEX]
 
I

ivory

Let [TEX]\blue x,y,z[/TEX] be positive real number such that [TEX]\blue xyz=x+y+z+2[/TEX].Prove that

[TEX]\blue (1)xy+yz+xz \ge 2(x+y+z)[/TEX]

[TEX]\blue (2)\sqrt{x}+\sqrt{y}+\sqrt{z} \le \frac{3}{2}\sqrt{xyz}[/TEX]
1)
c1:[TEX]a^2=\frac{1}{xy}, b^2=\frac{1}{yz}, c^2=\frac{1}{zx}[/TEX]
[TEX]a^2+b^2+c^2+2abc=1[/TEX]
------------------------
c2:
Tồn tại hai trong ba số cùng >=2, hoặc cùng=<2, giả sử là x,y
Ta có: [TEX](x-2)(y-2)\ge 0\leftright xy+4\ge 2(x+y)\leftright xy+zx+yz\ge (z+2)(x+y)-4[/TEX]
Chung minh [TEX](z+2)(x+y)-4\ge 2(x+y+z)[/TEX]
.....
2)
[TEX]\leftright a+b+c\le \frac{3}{2}\leftright cosA+cosB+cosC\le \frac{3}{2}[/TEX]
 
T

tranghh4

Áp dụng BDT [TEX]\frac{1}{x+y+z}[/TEX]\leq[TEX]\frac{1}{9}[/TEX].[TEX](\frac{1}{x}+\frac{1}{y}+\frac{1}{z})[/TEX].

ta có:

[TEX]\frac{ab}{a+3b+2c}[/TEX]\leq[TEX]\frac{ab}{9}[/TEX].[TEX](\frac{1}{b+c}+\frac{1}{a+c}+\frac{b+c})[/TEX]----(1)

Tương tự ta có:

[TEX]\frac{bc}{b+3c+2a}[/TEX]\leq[TEX]\frac{bc}{9}[/TEX].[TEX](\frac{1}{a+b}+\frac{1}{b+c}+\frac{1}{c+a}+\frac{b+c})[/TEX]-----(2)

[TEX]\frac{ca}{c+3a+2b}[/TEX]\leq[TEX](\frac{ca}{9}.(\frac{1}{a+b}+\frac{1}{b+c}+\frac{1}{c+a})[/TEX]---(3)

Công (1) (2) (3) ta dc ---> dpcm

Dấu bằng xảy ra \Leftrightarrowa=b=c

>> xin lỗi ,cái latex em đánh hơi lộn chút >>>
mấy anh chị thông cảm dùm ạ
ai ghải thích hộ mình chỗ này được hôk mình công lâj nhưng cũng chả ra cái gì hết.
 
T

tell_me_goobye

Cho a,b,c dương và [TEX]abc=2[/TEX]. Chứng minh:
[TEX]a^3+b^3+c^3\geq a\sqrt{b+c}+b\sqrt{c+a}+c\sqrt{a+b}[/TEX].


dễ có [TEX] \sum a\sqrt{b+c} [/TEX]\leq [TEX]\sqrt{2(a^2+b^2+c^2)(a+b+c)}[/TEX]
theo bdt chebyshev ta có
[TEX]\sqrt{2(a^2+b^2+c^2)(a+b+c)}[/TEX] \leq[TEX]\sqrt{6(a^3+b^3+c^3)}[/TEX]

bdt này CM nhờ 2 bdt sau:
[TEX]3(a^2+b^2+c^2)\geq(a+b+c)^2[/TEX]

[TEX] (a^2+b^2+c^2)^2 \leq(a+b+c)(a^3+b^3+c^3)[/TEX]

kết hợp với abc=2 [TEX]\Rightarrow[/TEX] dpcm
 
Last edited by a moderator:
T

tell_me_goobye

Lượng giác(hojoo lee)
[tex]a^2=tanx,b^2=tan^2y,c^2=tanz,d^2=tant[/tex]
co [TEX]\cos^2x+cos^2y+cos^2z+cos^2t=1[/TEX]
*[TEX]sin^2x=cos^2y+cos^2z+cos^2t\ge 3\sqrt[3]{(cosycoszcost)^2}[/TEX]
*[TEX]sin^2y\ge 3\sqrt[3]{(cosxcoszcost)^2}[/TEX]
*[TEX]sin^2z\ge 3\sqrt[3]{(cosxcosycost)^2}[/TEX]
*[TEX]sin^2t\ge 3\sqrt[3]{(cosxcosycosz)^2}[/TEX]

cách hay hơn nè

1=[TEX] \sum \frac{1}{a^4+1} = \sum \frac{\frac{1}{a^4}}{ \frac{1}{a^4} +1}[/TEX]

[TEX]\geq \frac{(\sum \frac{1}{a^2})^2}{\sum \frac{1}{a^4} +4}[/TEX]

[TEX]\Rightarrow \sum \frac{1}{a^4} +4 \geq (\sum \frac{1}{a^2})^2[/TEX]
[TEX]\Leftrightarrow2\geq \frac{1}{a^2b^2}+\frac{1}{a^2c^2}+\frac{1}{a^2d^2}+\frac{1}{b^2c^2}+\frac{1}{b^2d^2}+\frac{1}{c^2d^2} [/TEX]

Bđt này đúng theo AM-GM 6 số
 
T

trydan

[Trần Nam Dũng]

[Trần Nam Dũng] Cho a, b, c là các số thực dương thỏa mãn điều kiện
gif.latex
Chứng minh rằng
gif.latex


Vietnam, 2002​
 
T

tell_me_goobye

bài khó thế ai làm được (bài này dùng kĩ thuật cao siêu lắm)

BÀI NÀY NGON HƠN NÈ

cho a,b,c là các số thực không âm và a+b+c=1
tìm max [TEX](a^3+b^3)(b^3+c^3)(c^3+a^3)[/TEX]
 
N

namtuocvva18

Cho x,y,z duong. Chung minh:
[TEX]4(xy+yz+zx)\leq \sqrt{(x+y)(y+z)(z+x)}(\sqrt{x+y}+\sqrt{y+z}+\sqrt{z+x})[/TEX].
 
N

namtuocvva18

Cho a,b,c duong thoa man:
[TEX]\frac{1}{a+b+1}+\frac{1}{b+c+1}+\frac{1}{c+a+1}\ge 1[/TEX].
Chung minh: [TEX]a+b+c\geq ab+bc+ca[/TEX].
 
N

namtuocvva18

Cho x,y,z duong va [TEX]xy+yz+zx=1[/TEX]. Chung minh:
[TEX]\sqrt{x^4+x^2}+\sqrt{y^4+y^2}+\sqrt{z^4+z^2}\leq 1+x^2+y^2+z^2[/TEX].
 
N

namtuocvva18

Cho x,y,z duong. Chung minh:
[TEX]\frac{x\sqrt{x}}{\sqrt{z(z^2+xy)}}+\frac{y\sqrt{y}}{\sqrt{x(x^2+yz)}}+\frac{z\sqrt{z}}{\sqrt{y(y^2+zx)}}\geq \frac{3}{2}[/TEX].
 
N

namtuocvva18

Cho [TEX]0<a,b,c \leq 1[/TEX]. Chung minh:
[TEX](1+\frac{1}{abc})(a+b+c)\geq 3+\frac{1}{a}+\frac{1}{b}+\frac{1}{c}[/TEX].
 
N

namtuocvva18

6,Cho a,b,c khong am thoa man [TEX]a+b+c=1[/TEX]. Chug minh:
[TEX]1+6(a^3+b^3+c^3)\geq 5(a^2+b^2+c^2)[/TEX].


7,Cho x,y,z duong va [TEX]xy+yz+zx=1[/TEX]. Chung minh:
[TEX]\frac{x}{y(1+x^2)}+\frac{y}{z(1+y^2)}+\frac{z}{x(1+z^2)}\geq \frac{9}{4}[/TEX].
 
K

khuongchinh

bai nay la bai tren toan hoc tuoi tre ma em
dai khai la
nhan tung ra
(1-1/a)(1-1/b)>=0
em vit 3 kai tuong tu nhu the rui cong vao
day la puoc kho nhat rui, em tu lam tip di
 
Status
Không mở trả lời sau này.
Top Bottom